Q20

 
hyewonkim89
Thanks Received: 5
Atticus Finch
Atticus Finch
 
Posts: 122
Joined: December 17th, 2012
 
 
trophy
Most Thankful
trophy
First Responder
 

Q20

by hyewonkim89 Tue Nov 26, 2013 10:58 am

What makes (E) better than (D)?

Is resolving an apparent paradox only applicable for Passage A?

Also, does the example shown in Passage A count as data?
 
eshire89
Thanks Received: 0
Vinny Gambini
Vinny Gambini
 
Posts: 0
Joined: September 02nd, 2013
 
 
 

Re: Q20

by eshire89 Wed Dec 04, 2013 8:53 pm

I agree. I chose D over E, and I see the case for E, but I am not sure why it is stronger than E
 
christine.defenbaugh
Thanks Received: 585
Atticus Finch
Atticus Finch
 
Posts: 536
Joined: May 17th, 2013
 
This post thanked 3 times.
 
 

Re: Q20

by christine.defenbaugh Sat Dec 07, 2013 2:43 am

Great question you two - (D) is very tempting here.

On an overlap question, it's critical to be very specific about the support for the answer in each passage.

The primary temptation of (D) stems from the fact that Passage A sets up the issue as a paradox. It's easy to allow this perspective to frame the issue for both passages - and that's exactly what the LSAT writers expect you to do. But Passage B never treats the situation as a paradox at all. That author dives immediately into a discussion of why relative wealth is important to us - but proposing an explanation for a phenomenon is not always the same as resolving a paradox.

However, both authors rely on data, supporting (E):

Passage A does not use the word data, but paragraph 2 refers indirectly to what seems to be data actually collected.

    "if we ask people....the 'required income' correlates"
    "we can also look at reported happiness over time"
    "job satisfaction....rises if wages rapidly increase"

Additionally, the study mentioned explicitly in line 25 and its results in line 32 are surely data!

Passage B's support is a bit easier to find: line 50 "the data show that..."


The Other Wrong Answers
(A)
Only Passage B makes reference to biological origins (and only then as a counterpoint the author disagrees with).
(B) Neither author uses wide belief as support for a conclusion.
(C) Neither author proposes accepting a claim for the sake of argument.


On overlap questions, be vigilant not to allow any framing of the issues from Passage A to bleed over into your assessment of Passage B!

I hope this helps clear things up a bit!
 
547494985
Thanks Received: 0
Vinny Gambini
Vinny Gambini
 
Posts: 6
Joined: March 17th, 2014
 
 
 

Re: Q20

by 547494985 Wed Jun 04, 2014 4:49 am

hi, I also chose D over E, but in retrospect, I see D has this apparent paradox, but even if Passage does have a paradox, its far from apparent.
 
asafezrati
Thanks Received: 6
Atticus Finch
Atticus Finch
 
Posts: 116
Joined: December 07th, 2014
 
This post thanked 1 time.
 
 

Re: Q20

by asafezrati Fri Sep 11, 2015 10:52 pm

C was somewhat attractive simply because I didn't understand what is the meaning behind it.
Can this be an appropriate example of this method (accept a claim for the sake of argument)?:
"It's obvious that dogs are not hot blooded creature. Even if we accept that they are mammals, my 3 year old sister has conducted a thorough study showing that dogs' body temperature is always slightly below the absolute zero."
User avatar
 
tommywallach
Thanks Received: 468
Atticus Finch
Atticus Finch
 
Posts: 1041
Joined: August 11th, 2009
 
 
 

Re: Q20

by tommywallach Fri Sep 18, 2015 8:14 pm

I'm afraid I don't totally understand your question. And though I always respect a good example, yours is a little too silly to prove much of a point. That said, I suppose it would technically fit the model of accepting one half of an argument in order to disprove the overall argument.

-t
Tommy Wallach
Manhattan LSAT Instructor
twallach@manhattanprep.com
Image
 
andrewgong01
Thanks Received: 61
Atticus Finch
Atticus Finch
 
Posts: 289
Joined: October 31st, 2016
 
 
 

Re: Q20

by andrewgong01 Thu May 25, 2017 6:01 pm

For Choice "C" what made it tempting for me and still not allowing me to rule it out categorically is that Passage B accepts that the study was conducted and does not dispute the data collected. In fact, he/she said it was valid and accepted it to argue his/her point that to explain the data we need to appeal to the fact that it is "success" that drives up satisfaction. Passage A seems to also accept the same claim from the research as it is used as an example and even says "And Indeed" on line 34 so it seems like A also accepts it to advance A's proposition, namely, people have a need to be wealthier than others based of the wealth of the "average" person
User avatar
 
ohthatpatrick
Thanks Received: 3806
Atticus Finch
Atticus Finch
 
Posts: 4661
Joined: April 01st, 2011
 
This post thanked 1 time.
 
 

Re: Q20

by ohthatpatrick Fri May 26, 2017 4:07 pm

Accepting the results of a study is not what we mean by "accept a claim for the sake of argument".

We mean saying something like,
"Is Lebron the best basketball player ever? Let's say, for the sake of argument, that Lebron is the best ACTIVE player. That still doesn't mean he's the best player ever, because some retired players like Jordan and Wilt might still be better than Lebron."

We could maybe say that Psg B does this sort of thing in line 42-44.

Psg A doesn't do this sort of thing.
 
bobjon1259
Thanks Received: 0
Vinny Gambini
Vinny Gambini
 
Posts: 16
Joined: November 27th, 2017
 
 
 

Re: Q20

by bobjon1259 Fri Feb 01, 2019 11:17 am

ohthatpatrick Wrote:Accepting the results of a study is not what we mean by "accept a claim for the sake of argument".

We mean saying something like,
"Is Lebron the best basketball player ever? Let's say, for the sake of argument, that Lebron is the best ACTIVE player. That still doesn't mean he's the best player ever, because some retired players like Jordan and Wilt might still be better than Lebron."

We could maybe say that Psg B does this sort of thing in line 42-44.

Psg A doesn't do this sort of thing.


My issue with the credited response is its liberal use of the term "assert." I can see why (E) is the best answer, but I'm really appalled by the fact that the test writers believe that the author of Passage A asserted that his/her position is supported by the data. The definition of assert is "to state a fact or belief confidently or forcefully." Indeed, the author of Passage A incorporates data to support his/her position, but by no means asserts that his/her position is supported by the data. Please let me know your thoughts, Patrick.
User avatar
 
ohthatpatrick
Thanks Received: 3806
Atticus Finch
Atticus Finch
 
Posts: 4661
Joined: April 01st, 2011
 
This post thanked 1 time.
 
 

Re: Q20

by ohthatpatrick Wed Feb 06, 2019 3:52 pm

I feel your pain.

That verb "assert" should not be used interchangeably with "imply / suggest / convey", etc.

It makes it specifically sound like an author articulated this sentiment: "my position is supported by data".

This is one of those frustrating "best available" correct answers. Nowadays, in RC, I find myself at least a few times in every section needing to say,
"Okay, Patrick ... you're picking choice ____ . Yes, it's not adequately supported by the text. But it's still better supported than the alternatives."

The stuff in psg A I would point to would be this:

line 10 puts out a position, and then the author says
- "for example, if we ask people .."
- "We can also look at reported happiness over time .."

LSAT could argue that saying "for example" or "we can also look at" is an assertion of support.

line 34 also puts out a position: "how people compare to their reference group is crucial for happiness" ... and then asserts support by again saying, "In East Germany, for example ..."

Pretend this was my argument:
Brenda clearly likes cheese. We can see this by looking over her past orders from GrubHub.

Is it fair to say in that argument that the author "asserted that her position on Brenda's affinity for cheese is supported by online food orders"?

Yes, I guess so ... because saying "We can see this by looking at ...." is an assertion of support.
 
ReginaP412
Thanks Received: 0
Vinny Gambini
Vinny Gambini
 
Posts: 15
Joined: June 23rd, 2020
 
 
 

Re: Q20

by ReginaP412 Mon Dec 07, 2020 12:33 pm

I'm a little unclear as to why Passage B did not contain a paradox (D)?

"This theory may sound good... but it is not the explanation best supported..."

Doesn't this point to a paradox that the passage then goes to explain?
 
Laura Damone
Thanks Received: 94
Atticus Finch
Atticus Finch
 
Posts: 468
Joined: February 17th, 2011
 
 
 

Re: Q20

by Laura Damone Tue Dec 08, 2020 7:10 pm

Hi!

I think my fellow instructor Christine Defenbaugh did a great job explaining this in her post, which was, interestingly, posted exactly 7 years to the day before yours, Regina! Scroll up this thread until you see her bold red letters!

To expand upon her post a bit, just because something requires an explanation doesn't make it a paradox. Neither does the fact that one explanation may sound good at first but is ultimately not as strong as a second explanation. A paradox is something that either is, or seems, self-contradictory.

Part of your trouble might be that you'll frequently see "defying expectation" as part of the definition of "paradox." This definition on its own, however, is a little loose for the LSAT. I agree that the part of passage B you cited has a bit of that "defying expectation" flavor: something sounds good at first but doesn't prove to be that good after all. But we didn't actually have a reason to expect the first explanation to be good. It just sounded good. So when it turns out not to be that good, it doesn't actually defy our expectation.

Hope this helps!
Laura Damone
LSAT Content & Curriculum Lead | Manhattan Prep